+7 495 120-13-73 | 8 800 500-97-74

(для регионов бесплатно)

Содержание

Параллельное соединение проводников — Технарь

Другой способ соединения проводников, применяемый на практике, называется параллельным. На рисунке 267, а изображено параллельное соединение двух электрических ламп, а на рисунке 267, б — схема этого соединения. Если в этой цепи выключить одну лампу, то другая будет продолжать гореть.

При параллельном соединении все проводники одним своим концом присоединяются к одной точке цепи А, а вторым к другой точке В (рис. 267, б). Поэтому напряжение на концах всех параллельно соединенных проводников одно и то же. Изображенные на рисунке 267, а лампы горят при одинаковом напряжении.

В точке В (рис. 267, б) электрический ток I разветвляется на два тока I, и I2, сходящиеся вновь в точке А, подобно тому как изображенный на рисунке 268 поток воды в реке распределяется по двум каналам, сходящимся затем вновь.

Понятно, что

I = I1 + I2

т. е. сила тока в не разветвленной части цепи равна сумме сил токов в отдельных параллельно соединенных проводниках.

При параллельном соединении как бы увеличивается площадь поперечного сечения проводника. Поэтому общее сопротивление цепи уменьшается и становится меньше сопротивления каждого из проводников, входящих в цепь. Так, например, сопротивление цепи, состоящей из двух одинаковых ламп (рис. 267, а), в два раза меньше сопротивления одной лампы:

R = R1/2

Участок цепи, состоящий из n параллельно соединенных проводников с одинаковым сопротивлением, можно рассматривать как один проводник, площадь сечения которого в n раз больше площади сечения одного проводника той же длины. Во столько же раз будет меньше и сопротивление этого участка, т, е.

R = R1/n

Сложнее рассчитывается сопротивление цепи, состоящей из нескольких проводников с разным сопротивлением. В этом случае надо складывать не сопротивления проводников, а величины, обратные сопротивлениям:

1/R = 1/R1 + 1/R2

Пример 1. В осветительную цепь включены параллельно четыре лампы сопротивлением 120 Ом каждая. Найти общее сопротивление участка цепи.

Пример 2. Участок цепи состоит из двух параллельно соединенных проводников сопротивлением R1 = 3 Ом, R2 = 6 Ом. Найти сопротивление этого участка цепи.

В одну и ту же электрическую цепь параллельно могут быть включены самые различные потребители электрической энергии. На рисунке 269

показано параллельное включение электрических ламп, нагревательных приборов и электродвигателя.

Параллельно включаемые в данную сеть потребители должны быть рассчитаны на одно и то же напряжение, равное напряжению в сети.

Напряжение в сети, используемое у нас для освещения и в бытовых приборах, бывает 127 и 220 В. Поэтому электрические лампы и различные бытовые электрические приборы изготовляют на 127 и 220 В. В практике часто применяется смешанное (последовательное и параллельное) соединение проводников,

Вопросы. 1. Какое соединение проводников называют параллельным? Изобразите его на схеме. 2. Какая из электрических величин одинакова для всех проводников, соединенных параллельно?

3. Как выражается сила тока в цепи до ее разветвления через силы токов в отдельных ветвях разветвления? 4. Во сколько раз сопротивление участка цепи, состоящего из двух одинаковых проводников, соединенных параллельно, меньше сопротивления одного проводника? 5. Как включают электрические лампы и бытовые электрические приборы в сеть? 6. Какие напряжения используют для освещения и бытовых нужд?

Упражнения. 1. Два проводника сопротивлением 10 и 15 Ом соединены параллельно. Найдите полное сопротивление этого участка. 2. Два проводника сопротивлением 4 и 8 Ом соединены параллельно

. Напряжение на проводниках 4 В. Найдите силу тока в каждом проводнике и в общей цепи.

Задание

Основываясь на законе Ома для участка цепи и его следствиях, докажите, что сопротивление R участка цепи, состоящего из двух проводников сопротивлением R1 и R2, соединенных параллельно, рассчитывается по формуле: 1/R = 1/R1 + 1/R2, или R = R1*R2/R1+R2.

Общее сопротивление при параллельном соединении проводников формула

Проводники в электрических цепях могут соединяться последовательно и параллельно.

При последовательном соединении проводников (рис. 1.9.1) сила тока во всех проводниках одинакова:

Рисунок 1.9.1.

По закону Ома, напряжения и на проводниках равны

Общее напряжение на обоих проводниках равно сумме напряжений 1 и 2:

где – электрическое сопротивление всей цепи. Отсюда следует:

При последовательном соединении полное сопротивление цепи равно сумме сопротивлений отдельных проводников.

Этот результат справедлив для любого числа последовательно соединенных проводников.

При параллельном соединении (рис. 1.9.2) напряжения 1 и 2 на обоих проводниках одинаковы:

Сумма токов 1 + 2, протекающих по обоим проводникам, равна току в неразветвленной цепи:

Этот результат следует из того, что в точках разветвления токов (узлы и ) в цепи постоянного тока не могут накапливаться заряды. Например, к узлу за время Δ подтекает заряд Δ, а утекает от узла за то же время заряд

1Δ + 2Δ. Следовательно, = 1 + 2.

Рисунок 1.9.2.

Записывая на основании закона Ома

где – электрическое сопротивление всей цепи, получим

При параллельном соединении проводников величина, обратная общему сопротивлению цепи, равна сумме величин, обратных сопротивлениям параллельно включенных проводников.

Этот результат справедлив для любого числа параллельно включенных проводников.

Формулы для последовательного и параллельного соединения проводников позволяют во многих случаях рассчитывать сопротивление сложной цепи, состоящей из многих резисторов. На рис. 1.9.3 приведен пример такой сложной цепи и указана последовательность вычислений.

Рисунок 1.9.3.

Следует отметить, что далеко не все сложные цепи, состоящие из проводников с различными сопротивлениями, могут быть рассчитаны с помощью формул для последовательного и параллельного соединения. На рис. 1.9.4 приведен пример электрической цепи, которую нельзя рассчитать указанным выше методом.

Рисунок 1.9.4.

Цепи, подобные изображенной на рис. 1.9.4, а также цепи с разветвлениями, содержащие несколько источников, рассчитываются с помощью правил Кирхгофа.

Сопротивление проводников. Параллельное и последовательное соединение проводников.

Электри́ческое сопротивле́ние — физическая величина, характеризующая свойства проводника препятствовать прохождению электрического тока и равная отношениюнапряжения на концах проводника к силе тока, протекающего по нему [1] . Сопротивление для цепей переменного тока и для переменных электромагнитных полей описывается понятиями импеданса и волнового сопротивления. Сопротивлением (резистором) также называют радиодеталь, предназначенную для введения в электрические цепи активного сопротивления.

Сопротивление (часто обозначается буквой R или r) считается, в определённых пределах, постоянной величиной для данного проводника; её можно рассчитать как

U — разность электрических потенциалов (напряжение) на концах проводника;

I — сила тока, протекающего между концами проводника под действием разности потенциалов.

При последовательном соединении проводников (рис. 1.9.1) сила тока во всех проводниках одинакова:

Последовательное соединение проводников

По закону Ома, напряжения U1 и U2 на проводниках равны

Общее напряжение U на обоих проводниках равно сумме напряжений U1 и U2:

где R – электрическое сопротивление всей цепи. Отсюда следует:

При последовательном соединении полное сопротивление цепи равно сумме сопротивлений отдельных проводников.

Этот результат справедлив для любого числа последовательно соединенных проводников.

При параллельном соединении (рис. 1.9.2) напряжения

U1 и U2 на обоих проводниках одинаковы:

Сумма токов I1 + I2, протекающих по обоим проводникам, равна току в неразветвленной цепи:

Этот результат следует из того, что в точках разветвления токов (узлы A и B) в цепи постоянного тока не могут накапливаться заряды. Например, к узлу A за время Δt подтекает заряд IΔt, а утекает от узла за то же время заряд I1Δt + I2Δt. Следовательно,I = I1 + I2.

Параллельное соединение проводников

Записывая на основании закона Ома

где R – электрическое сопротивление всей цепи, получим

При параллельном соединении проводников величина, обратная общему сопротивлению цепи, равна сумме величин, обратных сопротивлениям параллельно включенных проводников.

Этот результат справедлив для любого числа параллельно включенных проводников.

Формулы для последовательного и параллельного соединения проводников позволяют во многих случаях рассчитывать сопротивление сложной цепи, состоящей из многих резисторов. На рис. 1.9.3 приведен пример такой сложной цепи и указана последовательность вычислений.

Расчет сопротивления сложной цепи. Сопротивления всех проводников указаны вомах (Ом)

Следует отметить, что далеко не все сложные цепи, состоящие из проводников с различными сопротивлениями, могут быть рассчитаны с помощью формул для последовательного и параллельного соединения. На рис. 1.9.4 приведен пример электрической цепи, которую нельзя рассчитать указанным выше методом.

Пример электрической цепи, которая не сводится к комбинации последовательно и параллельно соединенных проводников

Течение тока в электрической цепи осуществляется по проводникам, в направлении от источника к потребителям. В большинстве подобных схем используются медные провода и электрические приемники в заданном количестве, обладающие различным сопротивлением. В зависимости выполняемых задач, в электрических цепях используется последовательное и параллельное соединение проводников. В некоторых случаях могут быть применены оба типа соединений, тогда этот вариант будет называться смешанным. Каждая схема имеет свои особенности и отличия, поэтому их нужно обязательно заранее учитывать при проектировании цепей, ремонте и обслуживании электрооборудования.

Последовательное соединение проводников

В электротехнике большое значение имеет последовательное и параллельное соединение проводников в электрической цепи. Среди них часто используется схема последовательного соединения проводников предполагающая такое же соединение потребителей. В этом случае включение в цепь выполняется друг за другом в порядке очередности. То есть, начало одного потребителя соединяется с концом другого при помощи проводов, без каких-либо ответвлений.

Свойства такой электрической цепи можно рассмотреть на примере участков цепи с двумя нагрузками. Силу тока, напряжение и сопротивление на каждом из них следует обозначить соответственно, как I1, U1, R1 и I2, U2, R2. В результате, получились соотношения, выражающие зависимость между величинами следующим образом: I = I1 = I2, U = U1 + U2, R = R1 + R2. Полученные данные подтверждаются практическим путем с помощью проведения измерений амперметром и вольтметром соответствующих участков.

Таким образом, последовательное соединение проводников отличается следующими индивидуальными особенностями:

  • Сила тока на всех участках цепи будет одинаковой.
  • Общее напряжение цепи составляет сумму напряжений на каждом участке.
  • Общее сопротивление включает в себя сопротивления каждого отдельного проводника.

Данные соотношения подходят для любого количества проводников, соединенных последовательно. Значение общего сопротивления всегда выше, чем сопротивление любого отдельно взятого проводника. Это связано с увеличением их общей длины при последовательном соединении, что приводит и к росту сопротивления.

Если соединить последовательно одинаковые элементы в количестве n, то получится R = n х R1, где R – общее сопротивление, R1 – сопротивление одного элемента, а n – количество элементов. Напряжение U, наоборот, делится на равные части, каждая из которых в n раз меньше общего значения. Например, если в сеть с напряжением 220 вольт последовательно включаются 10 ламп одинаковой мощности, то напряжение в любой из них составит: U1 = U/10 = 22 вольта.

Проводники, соединенные последовательно, имеют характерную отличительную особенность. Если во время работы отказал хотя-бы один из них, то течение тока прекращается во всей цепи. Наиболее ярким примером является елочная гирлянда, когда одна перегоревшая лампочка в последовательной цепи, приводит к выходу из строя всей системы. Для установления перегоревшей лампочки понадобится проверка всей гирлянды.

Параллельное соединение проводников

В электрических сетях проводники могут соединяться различными способами: последовательно, параллельно и комбинированно. Среди них параллельное соединение это такой вариант, когда проводники в начальных и конечных точках соединяются между собой. Таким образом, начала и концы нагрузок соединяются вместе, а сами нагрузки располагаются параллельно относительно друг друга. В электрической цепи могут содержаться два, три и более проводников, соединенных параллельно.

Если рассматривать последовательное и параллельное соединение, сила тока в последнем варианте может быть исследована с помощью следующей схемы. Берутся две лампы накаливания, обладающие одинаковым сопротивлением и соединенные параллельно. Для контроля к каждой лампочке подключается собственный амперметр. Кроме того, используется еще один амперметр, контролирующий общую силу тока в цепи. Проверочная схема дополняется источником питания и ключом.

После замыкания ключа нужно контролировать показания измерительных приборов. Амперметр на лампе № 1 покажет силу тока I1, а на лампе № 2 – силу тока I2. Общий амперметр показывает значение силы тока, равное сумме токов отдельно взятых, параллельно соединенных цепей: I = I1 + I2. В отличие от последовательного соединения, при перегорании одной из лампочек, другая будет нормально функционировать. Поэтому в домашних электрических сетях используется параллельное подключение приборов.

С помощью такой же схемы можно установить значение эквивалентного сопротивления. С этой целью в электрическую цепь добавляется вольтметр. Это позволяет измерить напряжение при параллельном соединении, сила тока при этом остается такой же. Здесь также имеются точки пересечения проводников, соединяющих обе лампы.

В результате измерений общее напряжение при параллельном соединении составит: U = U1 = U2. После этого можно рассчитать эквивалентное сопротивление, условно заменяющее все элементы, находящиеся в данной цепи. При параллельном соединении, в соответствии с законом Ома I = U/R, получается следующая формула: U/R = U1/R1 + U2/R2, в которой R является эквивалентным сопротивлением, R1 и R2 – сопротивления обеих лампочек, U = U1 = U2 – значение напряжения, показываемое вольтметром.

Следует учитывать и тот фактор, что токи в каждой цепи, в сумме составляют общую силу тока всей цепи. В окончательном виде формула, отражающая эквивалентное сопротивление будет выглядеть следующим образом: 1/R = 1/R1 + 1/R2. При увеличении количества элементов в таких цепях – увеличивается и число слагаемых в формуле. Различие в основных параметрах отличают друг от друга и источников тока, позволяя использовать их в различных электрических схемах.

Параллельное соединение проводников характеризуется достаточно малым значением эквивалентного сопротивления, поэтому сила тока будет сравнительно высокой. Данный фактор следует учитывать, когда в розетки включается большое количество электроприборов. В этом случае сила тока значительно возрастает, приводя к перегреву кабельных линий и последующим возгораниям.

Законы последовательного и параллельного соединения проводников

Данные законы, касающиеся обоих видов соединений проводников, частично уже были рассмотрены ранее.

Для более четкого их понимания и восприятия в практической плоскости, последовательное и параллельное соединение проводников, формулы следует рассматривать в определенной последовательности:

  • Последовательное соединение предполагает одинаковую силу тока в каждом проводнике: I = I1 = I2.
  • Закон ома параллельное и последовательное соединение проводников объясняет в каждом случае по-своему. Например, при последовательном соединении, напряжения на всех проводниках будут равны между собой: U1 = IR1, U2 = IR2. Кроме того, при последовательном соединении напряжение составляет сумму напряжений каждого проводника: U = U1 + U2 = I(R1 + R2) = IR.
  • Полное сопротивление цепи при последовательном соединении состоит из суммы сопротивлений всех отдельно взятых проводников, независимо от их количества.
  • При параллельном соединении напряжение всей цепи равно напряжению на каждом из проводников: U1 = U2 = U.
  • Общая сила тока, измеренная во всей цепи, равна сумме токов, протекающих по всем проводникам, соединенных параллельно между собой: I = I1 + I2.

Для того чтобы более эффективно проектировать электрические сети, нужно хорошо знать последовательное и параллельное соединение проводников и его законы, находя им наиболее рациональное практическое применение.

Смешанное соединение проводников

В электрических сетях как правило используется последовательное параллельное и смешанное соединение проводников, предназначенное для конкретных условий эксплуатации. Однако чаще всего предпочтение отдается третьему варианту, представляющему собой совокупность комбинаций, состоящих из различных типов соединений.

В таких смешанных схемах активно применяется последовательное и параллельное соединение проводников, плюсы и минусы которых обязательно учитываются при проектировании электрических сетей. Эти соединения состоят не только из отдельно взятых резисторов, но и довольно сложных участков, включающих в себя множество элементов.

Смешанное соединение рассчитывается в соответствии с известными свойствами последовательного и параллельного соединения. Метод расчета заключается в разбивке схемы на более простые составные части, которые считаются отдельно, а потом суммируются друг с другом.

Сопротивление проводников и их соединение

Последовательное и параллельное соединения в электротехнике — два основных способа соединения элементовэлектрической цепи. При последовательном соединении все элементы связаны друг с другом так, что включающий их участок цепи не имеет ни одного узла. При параллельном соединении все входящие в цепь элементы объединены двумяузлами и не имеют связей с другими узлами, если это не противоречит условию.

При последовательном соединении проводников сила тока во всех проводниках одинакова.

При параллельном соединении падение напряжения между двумя узлами, объединяющими элементы цепи, одинаково для всех элементов. При этом величина, обратная общему сопротивлению цепи, равна сумме величин, обратных сопротивлениям параллельно включенных проводников.

Последовательное соединение

При последовательном соединении проводников сила тока в любых частях цепи одна и та же: 

Полное напряжение в цепи при последовательном соединении, или напряжение на полюсах источника тока, равно сумме напряжений на отдельных участках цепи:

    1. Параллельное соединение

Сила тока в неразветвленной части цепи равна сумме сил токов в отдельных параллельно соединённых проводниках: 

Напряжение на участках цепи АВ и на концах всех параллельно соединённых проводников одно и то же: 

Для двух параллельно соединённых резисторов их общее сопротивление равно: .

    1. Зависимость сопротивления проводника от температуры

Удельное сопротивление, а следовательно, и сопротивление металлов, зависит от температуры, увеличиваясь с ее ростом. Температурная зависимость сопротивления проводника объясняется тем, что

  1. возрастает интенсивность рассеивания (число столкновений) носителей зарядов при повышении температуры;

  2. изменяется их концентрация при нагревании проводника.

Опыт показывает, что при не слишком высоких и не слишком низких температурах зависимости удельного сопротивления и сопротивления проводника от температуры выражаются формулами:

 ρt=ρ0(1+αt),

 Rt=R0(1+αt),

где ρ0ρt — удельные сопротивления вещества проводника соответственно при 0 °С и t °C; R0Rt — сопротивления проводника при 0 °С и t °С, α — температурный коэффициент сопротивления: измеряемый в СИ в Кельвинах в минус первой степени (К-1). Для металлических проводников эти формулы применимы начиная с температуры 140 К и выше.

Сверхпроводи́мость — свойство некоторых материалов обладать строго нулевым электрическим сопротивлением при достижении ими температуры ниже определённого значения (критическая температура). Известны несколько сотен соединений, чистых элементов, сплавов и керамик, переходящих в сверхпроводящее состояние. Сверхпроводимость —квантовое явление. Оно характеризуется также эффектом Мейснера, заключающемся в полном вытеснении магнитного поля из объема сверхпроводника. Существование этого эффекта показывает, что сверхпроводимость не может быть описана просто как идеальная проводимость в классическом понимании.

Последовательное и параллельное сопротивление проводников. Закон Ома. Соединение проводников

Одним из китов, на котором держатся многие понятия в электронике, является понятие последовательного и параллельного подключения проводников. Знать основные отличия указанных типов подключения просто необходимо. Без этого нельзя понять и прочитать ни одной схемы.

Основные принципы

Электрический ток движется по проводнику от источника к потребителю (нагрузке). Чаще всего в качестве проводника выбирается медный кабель. Связано это с требованием, которое предъявляется к проводнику: он должен легко высвобождать электроны.

Независимо от способа подключения, электрический ток двигается от плюса к минусу. Именно в этом направлении убывает потенциал. При этом стоит помнить, что провод, по котору идет ток, также обладает сопротивлением. Но его значение очень мало. Именно поэтому им пренебрегают. Сопротивление проводника принимают равным нулю. В том случае, если проводник обладает сопротивлением, его принято называть резистором.

Параллельное подключение

В данном случае элементы, входящие в цепь, объединены между собой двумя узлами. С другими узлами у них связей нет. Участки цепи с таким подключением принято называть ветвями. Схема параллельного подключения представлена на рисунке ниже.

Если говорить более понятным языком, то в данном случае все проводники одним концом соединены в одном узле, а вторым — во втором. Это приводит к тому, что электрический ток разделяется на все элементы. Благодаря этому увеличивается проводимость всей цепи.

При подключении проводников в цепь данным способом напряжение каждого из них будет одинаково. А вот сила тока всей цепи будет определяться как сумма токов, протекающих по всем элементам. С учетом закона Ома путем нехитрых математических расчетов получается интересная закономерность: величина, обратная общему сопротивлению всей цепи, определяется как сумма величин, обратных сопротивлениям каждого отдельного элемента. При этом учитываются только элементы, подключенные параллельно.

Последовательное подключение

В данном случае все элементы цепи соединены таким образом, что они не образуют ни одного узла. При данном способе подключения имеется один существенный недостаток. Он заключается в том, что при выходе из строя одного из проводников все последующие элементы работать не смогут. Ярким примером такой ситуации является обычная гирлянда. Если в ней перегорает одна из лампочек, то вся гирлянда перестает работать.

Последовательное подключение элементов отличается тем, что сила тока во всех проводниках равна. Что касается напряжения цепи, то оно равно сумме напряжения отдельных элементов.

В данной схеме проводники включаются в цепь поочередно. А это значит, что сопротивление всей цепи будет складываться из отдельных сопротивлений, характерных для каждого элемента. То есть общее сопротивление цепи равно сумме сопротивлений всех проводников. Эту же зависимость можно вывести и математическим способом, используя закон Ома.

Смешанные схемы

Бывают ситуации, когда на одной схеме можно увидеть одновременно последовательное и параллельное подключение элементов. В таком случае говорят о смешанном соединении. Расчет подобных схем проводится отдельно для каждой из группы проводников.

Так, чтобы определить общее сопротивление, необходимо сложить сопротивление элементов, подключенных параллельно, и сопротивление элементов с последовательным подключением. При этом последовательное подключение является доминантным. То есть его рассчитывают в первую очередь. И только после этого определяют сопротивление элементов с параллельным подключением.

Подключение светодиодов

Зная основы двух типов подключения элементов в цепи, можно понять принцип создания схем различных электроприборов. Рассмотрим пример. во многом зависит от напряжения источника тока.

При небольшом напряжении сети (до 5 В) светодиоды подключают последовательно. Снизить уровень электромагнитных помех в данном случае поможет конденсатор проходного типа и линейные резисторы. Проводимость светодиодов увеличивают за счет использования системных модуляторов.

При напряжении сети 12 В может использоваться и последовательное, и параллельное подключение сети. В случае последовательного подключения используют импульсные блоки питания. Если собирается цепь из трех светодиодов, то можно обойтись без усилителя. Но если цепь будет включать большее количество элементов, то усилитель необходим.

Во втором случае, то есть при параллельном подключении, необходимо использование двух открытых резисторов и усилителя (с пропускной способностью выше 3 А). Причем первый резистор устанавливается перед усилителем, а второй — после.

При высоком напряжении сети (220 В) прибегают к последовательному подключению. При этом дополнительно используют операционные усилители и понижающие блоки питания.

Последовательное, параллельное и смешанное соединения резисторов. Значительное число приемников, включенных в электрическую цепь (электрические лампы, электронагревательные приборы и др.), можно рассматривать как некоторые элементы, имеющие определенное сопротивление. Это обстоятельство дает нам возможность при составлении и изучении электрических схем заменять конкретные приемники резисторами с определенными сопротивлениями. Различают следующие способы соединения резисторов (приемников электрической энергии): последовательное, параллельное и смешанное.

Последовательное соединение резисторов . При последовательном соединении нескольких резисторов конец первого резистора соединяют с началом второго, конец второго — с началом третьего и т. д. При таком соединении по всем элементам последовательной цепи проходит
один и тот же ток I.
Последовательное соединение приемников поясняет рис. 25, а.
.Заменяя лампы резисторами с сопротивлениями R1, R2 и R3, получим схему, показанную на рис. 25, б.
Если принять, что в источнике Ro = 0, то для трех последовательно соединенных резисторов согласно второму закону Кирхгофа можно написать:

E = IR 1 + IR 2 + IR 3 = I(R 1 + R 2 + R 3) = IR эк (19)

где R эк = R 1 + R 2 + R 3 .
Следовательно, эквивалентное сопротивление последовательной цепи равно сумме сопротивлений всех последовательно соединенных резисторов.Так как напряжения на отдельных участках цепи согласно закону Ома: U 1 =IR 1 ; U 2 = IR 2 , U 3 = IR з и в данном случае E = U, то длярассматриваемой цепи

U = U 1 + U 2 +U 3 (20)

Следовательно, напряжение U на зажимах источника равно сумме напряжений на каждом из последовательно включенных резисторов.
Из указанных формул следует также, что напряжения распределяются между последовательно соединенными резисторами пропорционально их сопротивлениям:

U 1: U 2: U 3 = R 1: R 2: R 3 (21)

т. е. чем больше сопротивление какого-либо приемника в последовательной цепи, тем больше приложенное к нему напряжение.

В случае если последовательно соединяются несколько, например п, резисторов с одинаковым сопротивлением R1, эквивалентное сопротивление цепи Rэк будет в п раз больше сопротивления R1, т. е. Rэк = nR1. Напряжение U1 на каждом резисторе в этом случае в п раз меньше общего напряжения U:

При последовательном соединении приемников изменение сопротивления одного из них тотчас же влечет за собой изменение напряжения на других связанных с ним приемниках. При выключении или обрыве электрической цепи в одном из приемников и в остальных приемниках прекращается ток. Поэтому последовательное соединение приемников применяют редко — только в том случае, когда напряжение источника электрической энергии больше номинального напряжения, на которое рассчитан потребитель. Например, напряжение в электрической сети, от которой питаются вагоны метрополитена, составляет 825 В, номинальное же напряжение электрических ламп, применяемых в этих вагонах, 55 В. Поэтому в вагонах метрополитена электрические лампы включают последовательно по 15 ламп в каждой цепи.
Параллельное соединение резисторов . При параллельном соединении нескольких приемников они включаются между двумя точками электрической цепи, образуя параллельные ветви (рис. 26, а). Заменяя

лампы резисторами с сопротивлениями R1, R2, R3, получим схему, показанную на рис. 26, б.
При параллельном соединении ко всем резисторам приложено одинаковое напряжение U. Поэтому согласно закону Ома:

I 1 =U/R 1 ; I 2 =U/R 2 ; I 3 =U/R 3 .

Ток в неразветвленной части цепи согласно первому закону Кирхгофа I = I 1 +I 2 +I 3 , или

I = U / R 1 + U / R 2 + U / R 3 = U (1/R 1 + 1/R 2 + 1/R 3) = U / R эк (23)

Следовательно, эквивалентное сопротивление рассматриваемой цепи при параллельном соединении трех резисторов определяется формулой

1/R эк = 1/R 1 + 1/R 2 + 1/R 3 (24)

Вводя в формулу (24) вместо значений 1/R эк, 1/R 1 , 1/R 2 и 1/R 3 соответствующие проводимости G эк, G 1 , G 2 и G 3 , получим: эквивалентная проводимость параллельной цепи равна сумме проводимостей параллельно соединенных резисторов :

G эк = G 1 + G 2 +G 3 (25)

Таким образом, при увеличении числа параллельно включаемых резисторов результирующая проводимость электрической цепи увеличивается, а результирующее сопротивление уменьшается.
Из приведенных формул следует, что токи распределяются между параллельными ветвями обратно пропорционально их электрическим сопротивлениям или прямо пропорционально их проводимостям. Например, при трех ветвях

I 1: I 2: I 3 = 1/R 1: 1/R 2: 1/R 3 = G 1 + G 2 + G 3 (26)

В этом отношении имеет место полная аналогия между распределением токов по отдельным ветвям и распределением потоков воды по трубам.
Приведенные формулы дают возможность определить эквивалентное сопротивление цепи для различных конкретных случаев. Например, при двух параллельно включенных резисторах результирующее сопротивление цепи

R эк =R 1 R 2 /(R 1 +R 2)

при трех параллельно включенных резисторах

R эк =R 1 R 2 R 3 /(R 1 R 2 +R 2 R 3 +R 1 R 3)

При параллельном соединении нескольких, например n, резисторов с одинаковым сопротивлением R1 результирующее сопротивление цепи Rэк будет в n раз меньше сопротивления R1, т.е.

R эк = R1 / n (27)

Проходящий по каждой ветви ток I1, в этом случае будет в п раз меньше общего тока:

I1 = I / n (28)

При параллельном соединении приемников, все они находятся под одним и тем же напряжением, и режим работы каждого из них не зависит от остальных. Это означает, что ток, проходящий по какому-либо из приемников, не будет оказывать существенного влияния на другие приемники. При всяком выключении или выходе из строя любого приемника остальные приемники остаются вклю-

ченными. Поэтому параллельное соединение имеет существенные преимущества перед последовательным, вследствие чего оно получило наиболее широкое распространение. В частности, электрические лампы и двигатели, предназначенные для работы при определенном (номинальном) напряжении, всегда включают параллельно.
На электровозах постоянного тока и некоторых тепловозах тяговые двигатели в процессе регулирования скорости движения нужно включать под различные напряжения, поэтому они в процессе разгона переключаются с последовательного соединения на параллельное.

Смешанное соединение резисторов . Смешанным соединением называется такое соединение, при котором часть резисторов включается последовательно, а часть — параллельно. Например, в схеме рис. 27, а имеются два последовательно включенных резистора сопротивлениями R1 и R2, параллельно им включен резистор сопротивлением Rз, а резистор сопротивлением R4 включен последовательно с группой резисторов сопротивлениями R1, R2 и R3.
Эквивалентное сопротивление цепи при смешанном соединении обычно определяют методом преобразования, при котором сложную цепь последовательными этапами преобразовывают в простейшую. Например, для схемы рис. 27, а вначале определяют эквивалентное сопротивление R12 последовательно включенных резисторов с сопротивлениями R1 и R2: R12 = R1 + R2. При этом схема рис. 27, а заменяется эквивалентной схемой рис. 27, б. Затем определяют эквивалентное сопротивление R123 параллельно включенных сопротивлений и R3 по формуле

R 123 =R 12 R 3 /(R 12 +R 3)=(R 1 +R 2)R 3 /(R 1 +R 2 +R 3).

При этом схема рис. 27, б заменяется эквивалентной схемой рис. 27, в. После этого находят эквивалентное сопротивление всей цепи суммированием сопротивления R123 и последовательно включенного с ним сопротивления R4:

R эк = R 123 + R 4 = (R 1 + R 2) R 3 / (R 1 + R 2 + R 3) + R 4

Последовательное, параллельное и смешанное соединения широко применяют для изменения сопротивления пусковых реостатов при пуске э. п. с. постоянного тока.

Проверим справедливость показанных здесь формул на простом эксперименте.

Возьмём два резистора МЛТ-2 на 3 и 47 Ом и соединим их последовательно. Затем измерим общее сопротивление получившейся цепи цифровым мультиметром. Как видим оно равно сумме сопротивлений резисторов, входящих в эту цепочку.


Замер общего сопротивления при последовательном соединении

Теперь соединим наши резисторы параллельно и замерим их общее сопротивление.


Измерение сопротивления при параллельном соединении

Как видим, результирующее сопротивление (2,9 Ом) меньше самого меньшего (3 Ом), входящего в цепочку. Отсюда вытекает ещё одно известное правило, которое можно применять на практике:

При параллельном соединении резисторов общее сопротивление цепи будет меньше наименьшего сопротивления, входящего в эту цепь.

Что ещё нужно учитывать при соединении резисторов?

Во-первых, обязательно учитывается их номинальная мощность. Например, нам нужно подобрать замену резистору на 100 Ом и мощностью 1 Вт . Возьмём два резистора по 50 Ом каждый и соединим их последовательно. На какую мощность рассеяния должны быть рассчитаны эти два резистора?

Поскольку через последовательно соединённые резисторы течёт один и тот же постоянный ток (допустим 0,1 А ), а сопротивление каждого из них равно 50 Ом , тогда мощность рассеивания каждого из них должна быть не менее 0,5 Вт . В результате на каждом из них выделится по 0,5 Вт мощности. В сумме это и будет тот самый 1 Вт .

Данный пример достаточно грубоват. Поэтому, если есть сомнения, стоит брать резисторы с запасом по мощности.

Подробнее о мощности рассеивания резистора читайте .

Во-вторых, при соединении стоит использовать однотипные резисторы, например, серии МЛТ. Конечно, нет ничего плохого в том, чтобы брать разные. Это лишь рекомендация.

Причем это могут быть не только проводники, но и конденсаторы. Здесь важно не запутаться в том, как выглядит каждое из них на схеме. А уже потом применять конкретные формулы. Их, кстати, нужно помнить наизусть.

Как различить эти два соединения?

Внимательно посмотрите на схему. Если провода представить как дорогу, то машины на ней будут играть роль резисторов. На прямой дороге без каких-либо разветвлений машины едут одна за другой, в цепочку. Так же выглядит и последовательное соединение проводников. Дорога в этом случае может иметь неограниченное количество поворотов, но ни одного перекрестка. Как бы ни виляла дорога (провода), машины (резисторы) всегда будут расположены друг за другом, по одной цепочке.

Совсем другое дело, если рассматривается параллельное соединение. Тогда резисторы можно сравнить со спортсменами на старте. Они стоят каждый на своей дорожке, но направление движения у них одинаковое, и финиш в одном месте. Так же и резисторы — у каждого из них свой провод, но все они соединены в некоторой точке.

Формулы для силы тока

О ней всегда идет речь в теме «Электричество». Параллельное и последовательное соединение по-разному влияют на величину в резисторах. Для них выведены формулы, которые можно запомнить. Но достаточно просто запомнить смысл, который в них вкладывается.

Так, ток при последовательном соединении проводников всегда одинаков. То есть в каждом из них значение силы тока не отличается. Провести аналогию можно, если сравнить провод с трубой. В ней вода течет всегда одинаково. И все препятствия на ее пути будут сметаться с одной и той же силой. Так же с силой тока. Поэтому формула общей силы тока в цепи с последовательным соединением резисторов выглядит так:

I общ = I 1 = I 2

Здесь буквой I обозначена сила тока. Это общепринятое обозначение, поэтому его нужно запомнить.

Ток при параллельном соединении уже не будет постоянной величиной. При той же аналогии с трубой получается, что вода разделится на два потока, если у основной трубы будет ответвление. То же явление наблюдается с током, когда на его пути появляется разветвление проводов. Формула общей силы тока при :

I общ = I 1 + I 2

Если разветвление составлено из проводов, которых больше двух, то в приведенной формуле на такое же количество станет больше слагаемых.

Формулы для напряжения

Когда рассматривается схема, в которой выполнено соединение проводников последовательно, то напряжение на всем участке определяется суммой этих величин на каждом конкретном резисторе. Сравнить эту ситуацию можно с тарелками. Удержать одну из них легко получится одному человеку, вторую рядом он тоже сможет взять, но уже с трудом. Держать в руках три тарелки рядом друг с другом одному человеку уже не удастся, потребуется помощь второго. И так далее. Усилия людей складываются.

Формула для общего напряжения участка цепи с последовательным соединением проводников выглядит так:

U общ = U 1 + U 2 , где U — обозначение, принятое для

Другая ситуация складывается, если рассматривается Когда тарелки ставятся друг на друга, их по-прежнему может удержать один человек. Поэтому складывать ничего не приходится. Такая же аналогия наблюдается при параллельном соединении проводников. Напряжение на каждом из них одинаковое и равно тому, которое на всех них сразу. Формула общего напряжения такая:

U общ = U 1 = U 2

Формулы для электрического сопротивления

Их уже можно не запоминать, а знать формулу закона Ома и из нее выводить нужную. Из указанного закона следует, что напряжение равно произведению силы тока и сопротивления. То есть U = I * R, где R — сопротивление.

Тогда формула, с которой нужно будет работать, зависит от того, как выполнено соединение проводников:

  • последовательно, значит, нужно равенство для напряжения — I общ * R общ = I 1 * R 1 + I 2 * R 2;
  • параллельно необходимо пользоваться формулой для силы тока — U общ / R общ = U 1 / R 1 + U 2 / R 2 .

Далее следуют простые преобразования, которые основываются на том, что в первом равенстве все силы тока имеют одинаковое значение, а во втором — напряжения равны. Значит, их можно сократить. То есть получаются такие выражения:

  1. R общ = R 1 + R 2 (для последовательного соединения проводников).
  2. 1 / R общ = 1 / R 1 + 1 / R 2 (при параллельном соединении).

При увеличении числа резисторов, которые включены в сеть, изменяется количество слагаемых в этих выражениях.

Стоит отметить, что параллельное и последовательное соединение проводников по-разному влияют на общее сопротивление. Первое из них уменьшает сопротивление участка цепи. Причем оно оказывается меньше самого маленького из использованных резисторов. При последовательном соединении все логично: значения складываются, поэтому общее число всегда будет самым большим.

Работа тока

Предыдущие три величины составляют законы параллельного соединения и последовательного расположения проводников в цепи. Поэтому их знать нужно обязательно. Про работу и мощность необходимо просто запомнить базовую формулу. Она записывается так: А = I * U * t , где А — работа тока, t — время его прохождения по проводнику.

Для того чтобы определить общую работу при последовательном соединении нужно заменить в исходном выражении напряжение. Получится равенство: А = I * (U 1 + U 2) * t, раскрыв скобки в котором получится, что работа на всем участке равна их сумме на каждом конкретном потребителе тока.

Аналогично идет рассуждение, если рассматривается схема параллельного соединения. Только заменять полагается силу тока. Но результат будет тот же: А = А 1 + А 2 .

Мощность тока

При выведении формулы для мощности (обозначение «Р») участка цепи опять нужно пользоваться одной формулой: Р = U * I. После подобных рассуждений получается, что параллельное и последовательное соединение описываются такой формулой для мощности: Р = Р 1 + Р 2 .

То есть, как бы ни были составлены схемы, общая мощность будет складываться из тех, которые задействованы в работе. Именно этим объясняется тот факт, что нельзя включать в сеть квартиры одновременно много мощных приборов. Она просто не выдержит такой нагрузки.

Как влияет соединение проводников на ремонт новогодней гирлянды?

Сразу же после того, как перегорит одна из лампочек, станет ясно, как они были соединены. При последовательном соединении не будет светиться ни одна из них. Это объясняется тем, что пришедшая в негодность лампа создает разрыв в цепи. Поэтому нужно проверить все, чтобы определить, какая перегорела, заменить ее — и гирлянда станет работать.

Если в ней используется параллельное соединение, то она не перестает работать при неисправности одной из лампочек. Ведь цепь не будет полностью разорвана, а только одна параллельная часть. Чтобы отремонтировать такую гирлянду, не нужно проверять все элементы цепи, а только те, которые не светятся.

Что происходит с цепью, если в нее включены не резисторы, а конденсаторы?

При их последовательном соединении наблюдается такая ситуация: заряды от плюсов источника питания поступают только на внешние обкладки крайних конденсаторов. Те, что находятся между ними, просто передают этот заряд по цепочке. Этим объясняется то, что на всех обкладках появляются одинаковые заряды, но имеющие разные знаки. Поэтому электрический заряд каждого конденсатора, соединенного последовательно, можно записать такой формулой:

q общ = q 1 = q 2 .

Для того чтобы определить напряжение на каждом конденсаторе, потребуется знание формулы: U = q / С. В ней С — емкость конденсатора.

Общее напряжение подчиняется тому же закону, который справедлив для резисторов. Поэтому, заменив в формуле емкости напряжение на сумму, мы получим, что общую емкость приборов нужно вычислять по формуле:

С = q / (U 1 + U 2).

Упростить эту формулу можно, перевернув дроби и заменив отношение напряжения к заряду емкостью. Получается такое равенство: 1 / С = 1 / С 1 + 1 / С 2 .

Несколько по-другому выглядит ситуация, когда соединение конденсаторов — параллельное. Тогда общий заряд определяется суммой всех зарядов, которые накапливаются на обкладках всех приборов. А значение напряжения по-прежнему определяется по общим законам. Поэтому формула для общей емкости параллельно соединенных конденсаторов выглядит так:

С = (q 1 + q 2) / U.

То есть эта величина считается, как сумма каждого из использованных в соединении приборов:

С = С 1 + С 2.

Как определить общее сопротивление произвольного соединения проводников?

То есть такого, в котором последовательные участки сменяют параллельные, и наоборот. Для них по-прежнему справедливы все описанные законы. Только применять их нужно поэтапно.

Сперва полагается мысленно развернуть схему. Если представить ее сложно, то нужно нарисовать то, что получается. Объяснение станет понятнее, если рассмотреть его на конкретном примере (см. рисунок).

Ее удобно начать рисовать с точек Б и В. Их необходимо поставить на некотором удалении друг от друга и от краев листа. Слева к точке Б подходит один провод, а вправо направлены уже два. Точка В, напротив, слева имеет два ответвления, а после нее расположен один провод.

Теперь необходимо заполнить пространство между этими точками. По верхнему проводу нужно расположить три резистора с коэффициентами 2, 3 и 4, а снизу пойдет тот, у которого индекс равен 5. Первые три соединены последовательно. С пятым резистором они параллельны.

Оставшиеся два резистора (первый и шестой) включены последовательно с рассмотренным участком БВ. Поэтому рисунок можно просто дополнить двумя прямоугольниками по обе стороны от выбранных точек. Осталось применить формулы для расчета сопротивления:

  • сначала ту, которая приведена для последовательного соединения;
  • потом для параллельного;
  • и снова для последовательного.

Подобным образом можно развернуть любую, даже очень сложную схему.

Задача на последовательное соединение проводников

Условие. В цепи друг за другом подсоединены две лампы и резистор. Общее напряжение равно 110 В, а сила тока 12 А. Чему равно сопротивление резистора, если каждая лампа рассчитана на напряжение в 40 В?

Решение. Поскольку рассматривается последовательное соединение, формулы его законов известны. Нужно только правильно их применить. Начать с того, чтобы выяснить значение напряжения, которое приходится на резистор. Для этого из общего нужно вычесть два раза напряжение одной лампы. Получается 30 В.

Теперь, когда известны две величины, U и I (вторая из них дана в условии, так как общий ток равен току в каждом последовательном потребителе), можно сосчитать сопротивление резистора по закону Ома. Оно оказывается равным 2,5 Ом.

Ответ. Сопротивление резистора равно 2,5 Ом.

Задача на параллельное и последовательное

Условие. Имеются три конденсатора с емкостями 20, 25 и 30 мкФ. Определите их общую емкость при последовательном и параллельном соединении.

Решение. Проще начать с В этой ситуации все три значения нужно просто сложить. Таким образом, общая емкость оказывается равной 75 мкФ.

Несколько сложнее расчеты будут при последовательном соединении этих конденсаторов. Ведь сначала нужно найти отношения единицы к каждой из этих емкостей, а потом сложить их друг с другом. Получается, что единица, деленная на общую емкость, равна 37/300. Тогда искомая величина получается приблизительно 8 мкФ.

Ответ. Общая емкость при последовательном соединении 8 мкФ, при параллельном — 75 мкФ.

Если нам надо, чтобы электроприбор работал, мы должны подключить его к . При этом ток должен проходить через прибор и возвращаться вновь к источнику, то есть цепь должна быть замкнутой.

Но подключение каждого прибора к отдельному источнику осуществимо, в основном, в лабораторных условиях. В жизни же приходится иметь дело с ограниченным количеством источников и довольно большим количеством потребителей тока. Поэтому создают системы соединений, позволяющие нагрузить один источник большим количеством потребителей. Системы при этом могут быть сколь угодно сложными и разветвленными, но в их основе лежит всего два вида соединения: последовательное и параллельное соединение проводников. Каждый вид имеет свои особенности, плюсы и минусы. Рассмотрим их оба.

Последовательное соединение проводников

Последовательное соединение проводников – это включение в электрическую цепь нескольких приборов последовательно, друг за другом. Электроприборы в данном случае можно сравнить с людьми в хороводе, а их руки, держащие друг друга – это провода, соединяющие приборы. Источник тока в данном случае будет одним из участников хоровода.

Напряжение всей цепи при последовательном соединении будет равно сумме напряжений на каждом включенном в цепь элементе. Сила тока в цепи будет одинакова в любой точке. А сумма сопротивлений всех элементов составит общее сопротивление всей цепи. Поэтому последовательное сопротивление можно выразить на бумаге следующим образом:

I=I_1=I_2=⋯=I_n ; U=U_1+U_2+⋯+U_n ; R=R_1+R_2+⋯+R_n ,

Плюсом последовательного соединения является простота сборки, а минусом – то, что если один элемент выйдет из строя, то ток пропадет во всей цепи. В такой ситуации неработающий элемент будет подобен ключу в выключенном положении. Пример из жизни неудобства такого соединения наверняка припомнят все люди постарше, которые украшали елки гирляндами из лампочек.

Если в такой гирлянде выходила из строя хотя бы одна лампочка, приходилось перебирать их все, пока не найдешь ту самую, перегоревшую. В современных гирляндах эта проблема решена. В них используют специальные диодные лампочки, в которых при перегорании сплавляются вместе контакты, и ток продолжает беспрепятственно проходить дальше.

Параллельное соединение проводников

При параллельном соединении проводников все элементы цепи подключаются к одной и той же паре точек, можно назвать их А и В. К этой же паре точек подключают источник тока. То есть получается, что все элементы подключены к одинаковому напряжению между А и В. В то же время ток как бы разделяется на все нагрузки в зависимости от сопротивления каждой из них.

Параллельное соединение можно сравнить с течением реки, на пути которой возникла небольшая возвышенность. Вода в таком случае огибает возвышенность с двух сторон, а потом вновь сливается в один поток. Получается островок посреди реки. Так вот параллельное соединение – это два отдельных русла вокруг острова. А точки А и В – это места, где разъединяется и вновь соединяется общее русло реки.

Напряжение тока в каждой отдельной ветви будет равно общему напряжению в цепи. Общий ток цепи будет складываться из токов всех отдельных ветвей. А вот общее сопротивление цепи при параллельном соединении будет меньше сопротивления тока на каждой из ветвей. Это происходит потому, что общее сечение проводника между точками А и В как бы увеличивается за счет увеличения числа параллельно подключенных нагрузок. Поэтому общее сопротивление уменьшается. Параллельное соединение описывается следующими соотношениями:

U=U_1=U_2=⋯=U_n ; I=I_1+I_2+⋯+I_n ; 1/R=1/R_1 +1/R_2 +⋯+1/R_n ,

где I — сила тока, U- напряжение, R – сопротивление, 1,2,…,n – номера элементов, включенных в цепь.

Огромным плюсом параллельного соединения является то, что при выключении одного из элементов, цепь продолжает функционировать дальше. Все остальные элементы продолжают работать. Минусом является то, что все приборы должны быть рассчитаны на одно и то же напряжение. Именно параллельным образом устанавливают розетки сети 220 В в квартирах. Такое подключение позволяет включать различные приборы в сеть совершенно независимо друг от друга, и при выходе их строя одного из них, это не влияет на работу остальных.

Нужна помощь в учебе?

Предыдущая тема: Расчёт сопротивления проводников и реостаты: формулы
Следующая тема:&nbsp&nbsp&nbspРабота и мощность тока

Напряжение и сопротивление при последовательном соединении

Сопротивление проводников. Параллельное и последовательное соединение проводников.

Электри́ческое сопротивле́ние — физическая величина, характеризующая свойства проводника препятствовать прохождению электрического тока и равная отношениюнапряжения на концах проводника к силе тока, протекающего по нему [1] . Сопротивление для цепей переменного тока и для переменных электромагнитных полей описывается понятиями импеданса и волнового сопротивления. Сопротивлением (резистором) также называют радиодеталь, предназначенную для введения в электрические цепи активного сопротивления.

Сопротивление (часто обозначается буквой R или r) считается, в определённых пределах, постоянной величиной для данного проводника; её можно рассчитать как

U — разность электрических потенциалов (напряжение) на концах проводника;

I — сила тока, протекающего между концами проводника под действием разности потенциалов.

При последовательном соединении проводников (рис. 1.9.1) сила тока во всех проводниках одинакова:

Последовательное соединение проводников

По закону Ома, напряжения U1 и U2 на проводниках равны

Общее напряжение U на обоих проводниках равно сумме напряжений U1 и U2:

где R – электрическое сопротивление всей цепи. Отсюда следует:

При последовательном соединении полное сопротивление цепи равно сумме сопротивлений отдельных проводников.

Этот результат справедлив для любого числа последовательно соединенных проводников.

При параллельном соединении (рис. 1.9.2) напряжения U1 и U2 на обоих проводниках одинаковы:

Сумма токов I1 + I2, протекающих по обоим проводникам, равна току в неразветвленной цепи:

Этот результат следует из того, что в точках разветвления токов (узлы A и B) в цепи постоянного тока не могут накапливаться заряды. Например, к узлу A за время Δt подтекает заряд IΔt, а утекает от узла за то же время заряд I1Δt + I2Δt. Следовательно,I = I1 + I2.

Параллельное соединение проводников

Записывая на основании закона Ома

где R – электрическое сопротивление всей цепи, получим

При параллельном соединении проводников величина, обратная общему сопротивлению цепи, равна сумме величин, обратных сопротивлениям параллельно включенных проводников.

Этот результат справедлив для любого числа параллельно включенных проводников.

Формулы для последовательного и параллельного соединения проводников позволяют во многих случаях рассчитывать сопротивление сложной цепи, состоящей из многих резисторов. На рис. 1.9.3 приведен пример такой сложной цепи и указана последовательность вычислений.

Расчет сопротивления сложной цепи. Сопротивления всех проводников указаны вомах (Ом)

Следует отметить, что далеко не все сложные цепи, состоящие из проводников с различными сопротивлениями, могут быть рассчитаны с помощью формул для последовательного и параллельного соединения. На рис. 1.9.4 приведен пример электрической цепи, которую нельзя рассчитать указанным выше методом.

Пример электрической цепи, которая не сводится к комбинации последовательно и параллельно соединенных проводников

Во всех электрических схемах используются резисторы, представляющие собой элементы, с точно установленным значением сопротивления. Благодаря специфическим качествам этих устройств, становится возможной регулировка напряжения и силы тока на любых участках схемы. Данные свойства лежат в основе работы практически всех электронных приборов и оборудования. Так, напряжение при параллельном и последовательном соединении резисторов будет отличаться. Поэтому каждый вид соединения может применяться только в определенных условиях, чтобы та или иная электрическая схема могла в полном объеме выполнять свои функции.

Напряжение при последовательном соединении

При последовательном соединении два резистора и более соединяются в общую цепь таким образом, что каждый из них имеет контакт с другим устройством только в одной точке. Иначе говоря, конец первого резистора соединяется с началом второго, а конец второго – с началом третьего и т.д.

Особенностью данной схемы является прохождение через все подключенные резисторы одного и того же значения электрического тока. С возрастанием количества элементов на рассматриваемом участке цепи, течение электрического тока становится все более затрудненным. Это происходит из-за увеличения общего сопротивления резисторов при их последовательном соединении. Данное свойство отражается формулой: Rобщ = R1 + R2.

Распределение напряжения, в соответствии с законом Ома, осуществляется на каждый резистор по формуле: VRn = IRn x Rn. Таким образом, при увеличении сопротивления резистора, возрастает и падающее на него напряжение.

Напряжение при параллельном соединении

При параллельном соединении, включение резисторов в электрическую цепь выполняется таким образом, что все элементы сопротивлений подключаются друг к другу сразу обоими контактами. Одна точка, представляющая собой электрический узел, может соединять одновременно несколько резисторов.

Такое соединение предполагает течение отдельного тока в каждом резисторе. Сила этого тока находится в обратно пропорциональной зависимости с сопротивлением резистора. В результате, происходит увеличение общей проводимости данного участка цепи, при общем уменьшении сопротивления. В случае параллельного соединения резисторов с различным сопротивлением, значение общего сопротивления на этом участке всегда будет ниже самого маленького сопротивления отдельно взятого резистора.

На представленной схеме, напряжение между точками А и В представляет собой не только общее напряжение для всего участка, но и напряжение, поступающее к каждому отдельно взятому резистору. Таким образом, в случае параллельного соединения, напряжение, подаваемое ко всем резисторам, будет одинаковым.

В результате, напряжение при параллельном и последовательном соединении будет отличаться в каждом случае. Благодаря этому свойству, имеется реальная возможность отрегулировать данную величину на любом участке цепи.

Последовательное соединение резисторов

Последовательное соединениеэто соединение двух или более резисторов в форме цепи, в которой каждый отдельный резистор соединяется с другим отдельным резистором только в одной точке.

Общее сопротивление R

общ

При таком соединении, через все резисторы проходит один и тот же электрический ток. Чем больше элементов на данном участке электрической цепи, тем «труднее» току протекать через него. Следовательно, при последовательном соединении резисторов их общее сопротивление увеличивается, и оно равно сумме всех сопротивлений.

Напряжение при последовательном соединении

Напряжение при последовательном соединении распределяется на каждый резистор согласно закону Ома:

Т.е чем большее сопротивление резистора, тем большее напряжение на него падает.

Параллельное соединение резисторов

Параллельное соединениеэто соединение, при котором резисторы соединяются между собой обоими контактами. В результате к одной точке (электрическому узлу) может быть присоединено несколько резисторов.

Общее сопротивление R

общ

При таком соединении, через каждый резистор потечет отдельный ток. Сила данного тока будет обратно пропорциональна сопротивлению резистора. В результате общая проводимость такого участка электрической цепи увеличивается, а общее сопротивление в свою очередь уменьшается.

Таким образом, при параллельном подсоединении резисторов с разным сопротивлением, общее сопротивление будет всегда меньше значения самого маленького отдельного резистора.

Формула общей проводимости при параллельном соединении резисторов:

Формула эквивалентного общего сопротивления при параллельном соединении резисторов:

Для двух одинаковых резисторов общее сопротивление будет равно половине одного отдельного резистора:

Соответственно, для n одинаковых резисторов общее сопротивление будет равно значению одного резистора, разделенного на n.

Напряжение при параллельном соединении

Напряжение между точками A и B является как общим напряжением для всего участка цепи, так и напряжением, падающим на каждый резистор в отдельности. Поэтому при параллельном соединении на все резисторы упадет одинаковое напряжение.

Электрический ток при параллельном соединении

Через каждый резистор течет ток, сила которого обратно пропорциональна сопротивлению резистора. Для того чтобы узнать какой ток течет через определенный резистор, можно воспользоваться законом Ома:

Смешанное соединение резисторов

Смешанным соединением называют участок цепи, где часть резисторов соединяются между собой последовательно, а часть параллельно. В свою очередь, смешанное соединение бывает последовательного и параллельного типов.

Общее сопротивление R

общ

Для того чтобы посчитать общее сопротивление смешанного соединения:

  • Цепь разбивают на участки с только пареллельным или только последовательным соединением.
  • Вычисляют общее сопротивление для каждого отдельного участка.
  • Вычисляют общее сопротивление для всей цепи смешанного соединения.

Так это будет выглядеть для схемы 1:

Также существует более быстрый способ расчета общего сопротивления для смешанного соединения. Можно, в соответствии схеме, сразу записывать формулу следующим образом:

  • Если резисторы соединяются последоватеьно — складывать.
  • Если резисторы соединяются параллельно — использовать условное обозначение «||».
  • Подставлять формулу для параллельного соединения где стоит символ «||».

Так это будет выглядеть для схемы 1:

После подстановки формулы параллельного соединения вместо «||»:

Закон Ома для участка цепи. Сопротивление. Соединение проводников

Для каждого проводника — твёрдого, жидкого и газообразного — существует определённая зависимость силы тока от приложенной разности потенциалов на концах проводника.

Металлический проводник, подключенный к источнику тока является примером однородного участка цепи.

Немецкий физик Георг Симон Ом экспериментально изучил зависимость силы тока в металлических проводниках от напряжения, пришел к выводу: если состояние проводника с течением времени не меняется, а его температура постоянна, то для каждого проводника существует однозначная связь между I и U — это вольт-амперная характеристика. Зависимость силы тока в проводнике от напряжения, подаваемого на него, называют вольт-амперной характеристикой проводника.

Измеряя силу тока амперметром, а напряжение вольтметром, можно убедиться в том, что сила тока прямо пропорциональна напряжению.

Закон Ома для участка цепи: Сила тока на участке цепи прямо пропорциональна приложенному к нему напряжению U и обратно пропорциональна сопротивлению этого участка R.   

Электрическое сопротивление проводника

Основная электрическая характеристика проводника — сопротивление. От этой величины зависит сила тока в проводнике при заданном напряжении. Свойство проводника ограничивать силу тока в цепи, т. е. противодействовать электрическому току, называют электрическим сопротивлением проводника. 

На рисунке приведены графики вольт-амперных характеристик двух проводников. Очевидно, что сопротивление проводника, которому соответствует график 2, больше, чем сопротивление проводника, которому соответствует график 1. Сопротивление проводника не зависит от напряжения и силы тока.

Сопротивление однородного металлического проводника постоянного сечения зависит от его геометрических размеров, формы и вещества, из которого изготовлен проводник.

Единицу сопротивления проводника устанавливают на основе закона Ома и называют её омом (Ом).

Удельное сопротивление проводника  зависит от рода вещества и его состояния, например, температуры. Удельное сопротивление для определенного вещества имеет постоянное табличное значение.

Закон Ома — основа расчётов электрических цепей в электротехнике.

 К наиболее простым и часто встречающимся соединениям проводников относятся последовательное и параллельное соединения.

Эти правила можно применить для любого числа соединённых проводников:


В осветительной сети обычно поддерживается напряжение 220 В. На это напряжение рассчитаны приборы, потребляющие электрическую энергию. Поэтому параллельное соединение — самый распространённый способ соединения различных потребителей. В этом случае выход из строя одного прибора не отражается на работе остальных, тогда как при последовательном соединении выход из строя одного прибора размыкает цепь.

Резистор — элемент электрической цепи, характеризуемый только сопротивлением электрическому току. На схемах резистор обозначается прямоугольником: 

Реостат — прибор, служащий для регулировки и получения требуемой величины сопротивления. Обозначение на схемах: 

  

Резисторы                          Реoстат

Метод ключевых ситуаций на уроке физики «Параллельное и последовательное соединение проводников» в 8-м классе

Рассмотрены примеры использования метода ключевых ситуаций при изучении темы «Параллельное и последовательное соединение проводников» в 8 классе.

Ключевые моменты.

  1. Расчет сопротивления цепи при параллельном и последовательном соединении проводников.
  2. Распределение сил токов и напряжений при последовательном и параллельном соединении проводников.

Расчет сопротивления цепи при параллельном и последовательном соединении проводников

Первый уровень.

Имеется три проводника сопротивлением по 3 Ом каждый. Как надо соединить эти проводники, чтобы сопротивление цепи было максимальным? минимальным? Рассчитайте сопротивление для каждого случая.

Второй уровень.

Три одинаковых проводника соединены последовательно. Как изменится сила тока в цепи, если последовательно с ними подключить еще один проводник?

Как изменится сила тока, если к параллельно включенным проводникам добавить еще один?

Третий уровень.

Как получить сопротивления 16 Ом и 36 Ом, используя три одинаковых сопротивления по 24 Ом?

Или

Из одинаковых резисторов по 10 Ом требуется составить цепь сопротивлением 6 Ом. Какое наименьшее количество резисторов для этого потребуется?

Начертите схему цепи.

Методика решения задач

После рассмотрения закономерностей последовательного и параллельного соединения предлагается для устного решения ряд задач первого уровня.

Учитель: Рассчитайте сопротивление цепи, состоящей из двух проводников по 2 Ом, соединенных последовательно.

Ученики: При последовательном соединении проводников сопротивления складываются. Следовательно, ответ – 4 Ом.

Учитель: Каким станет сопротивление, если добавить еще один проводник 2 Ом? 10 Ом?

Ученики: 6 Ом, 14 Ом.

Если будем последовательно подключать еще проводники, как будет меняться общее сопротивление цепи?

Ученики: Увеличиваться.

Учитель: Попробуем обобщить результаты и сделать выводы. Если включаем последовательно n проводников сопротивлением R каждый, чему равно общее сопротивление цепи?

Ученики: Rобщ = nR.

Учитель: Как изменяется общее сопротивление последовательно включенных проводников при подключении каждого последующего проводника?

Ученики. Увеличивается.

Учитель: Заметим также, что общее сопротивление цепи при последовательном включении проводников будет больше каждого, даже самого большого.

Рассмотрим аналогичную задачу, но проводники соединены параллельно. Итак, два проводника по 2 Ом соединены параллельно. Каково общее сопротивление цепи?

Ученики: при параллельном соединении складываются величины обратные сопротивлениям. Следовательно, воспользовавшись формулой 1/R = 1/R1 + 1/R2, получаем 1 Ом.

Учитель: А если величина сопротивлений по 4 Ом?

Ученики: Воспользуемся этой же формулой. Общее сопротивление цепи – 2 Ом.

Учитель: Посмотрите на результаты и скажите, как рассчитать без данной формулы общее сопротивление параллельно включенных резисторов, если их величины одинаковы?

Ученики: Rобщ = R/n.

Учитель: А значит, если мы будем подключать параллельно еще резисторы, общее сопротивление будет…?

Ученики: Уменьшаться.

Учитель: Верно. А теперь добавим к нашим резисторам по 2 Ом параллельно в первом случае сопротивление 1/2 Ом, во втором — 1000 Ом. Что получим?

Ученики: Получаем в первом случае — 1/3 Ом, во втором — 1000/1001 Ом.

Учитель: Проанализируем результаты. При параллельном подключении маленького сопротивления общее сопротивление уменьшилось и стало меньше самого маленького. При подсоединении большого сопротивления общее все равно уменьшилось и его величина все равно меньше самого маленького.
Запомним наши выводы. Во-первых, при последовательном соединении одинаковых резисторов сопротивлением R общее сопротивление цепи nR, при параллельном включении — R/n. Во-вторых, при последовательном соединении общее сопротивление — больше самого большого, при параллельном – меньше самого маленького.

(В дальнейшем эти выводы пригодятся при рассмотрении причин короткого замыкания)

После этого можно предложить для устного или полу-устного решения задачи первого, второго и третьего уровней, помогая использовать при рассуждениях полученные знания.

Ученики: (Задача второго уровня) Если к последовательно подключить еще один проводник, то общее сопротивление увеличится: 4R → 5R.  I = U/R, следовательно, при неизменном напряжении сила тока уменьшится: I1 = U/4R → I2 = U/5R → сила тока уменьшится в 5/4 раз = 1,25 раз.  При параллельном соединении общее сопротивление уменьшиться: R/4 → R/5 → сила тока увеличится в 1,25 раз.

Задача третьего уровня.

Ученики: Максимальное и минимальное сопротивления, которые можно получить, используя данные резисторы – 72 Ом и 8 Ом соответственно. Значит, надо использовать оба вида соединений. Два параллельно включенных резистора дают 12 Ом + последовательно еще 24 Ом. (Рисуют схему). Два последовательно включенных дают сопротивление 48 Ом + параллельно с ними 24 Ом. Получаем 16 Ом (схема).

Учитель: В качестве домашнего задания подумайте, какие еще сопротивления можно получить. Используя данные резисторы.
Можно в качестве домашнего задания предложить вторую задачу.

Распределение сил токов и напряжений при последовательном и параллельном соединении проводников

Первый уровень.

Два резистора сопротивлением 3 Ом и 6 Ом соединены последовательно (параллельно). К концам цепи приложено напряжение 36 В. Найдите силу тока и напряжение на каждом резисторе.

Второй уровень.

Три резистора сопротивлением 3 Ом, 6 Ом и 18 Ом соединены последовательно (параллельно). К концам цепи приложено напряжение 36 В. Найдите силу тока и  напряжение на каждом резисторе.

Третий уровень.

К участку цепи приложено напряжение 6 В. Сопротивление каждого резистора 1 Ом. Сравните напряжения и силу тока в резисторах.

Методика разбора задач на уроке
  1. Учитель, познакомив с закономерностями последовательного и параллельного соединения, предлагает решить задачу первого уровня. Сначала задача решается с помощью формул: Rобщ = R1+ R2, I = U/ Rобщ, I = I1 = I2, U1 = IR1, U2 = IR2 (или U2 = U – U1).
  2. Затем учитель просит учеников предложить другой способ решения и подводит их к следующим рассуждениям: так как второе сопротивление в 2 раза больше, то напряжение на нем в два раза меньше, а их сумма известна и равна 36 В. Следовательно, если на первое сопротивление приходится одна часть напряжения, то на второе – две части, всего – три части. 36 В делим на три части. Получаем, что на каждую часть приходится 12 В. Следовательно, напряжение на первом резисторе равно 12 В, на втором – 24 В. То есть задача сводится к известной из курса математики «задаче на части».
  3. Задачу второго уровня можно решить двумя способами для закрепления. Ученикам предложить высказаться в пользу одного или другого метода.
  4. После предыдущих упражнений третью задачу можно решить устно: напряжение на резисторах 1 и 4 одинаково и равно 6 В. Следовательно, сила тока тоже одинакова и равна 6 А. Напряжение на среднем участке 6 В. Сопротивление среднего участка в два раза больше. Следовательно, сила тока в два раза меньше и равна 3 А. напряжения на резисторах 2 и 3 одинаковы и равны 3 В.
    Проверим: общее сопротивление резисторов 1 и 4 равно ½ Ом. Общее сопротивление резисторов 2 и 3 равно 2 Ом. Сопротивление всего участка 2/5 Ом. Сила тока на всем участке 15 А. Совпадает с полученным результатом 6А+6А+3А = 15 А.

Падение напряжения, параллельные проводники — таблицы проводов / определение сечения проводников

Расчет падения напряжения

Иногда необходимо рассчитать падение напряжения в установке. когда известны длина, размер провода и сила тока. Следующая формула может использоваться для определения падения напряжения на проводниках, используемых в однофазной сети. система:

E_D = 2 KIL / CM

где:

E_D = падение напряжения

K = Ом на милфут

I = ток

L = длина проводника в футах

CM = площадь жилы в круглой миле

===

ПРИМЕР:

Определить сопротивление в омах на мил при 75 ° C для медного проводника? Решение Используйте формулу R = R_ref

[1 + a (T x Tref)], где

R = Сопротивление проводника при температуре «T»

R_ref = Сопротивление проводника при эталонной температуре (в данном примере 20 ° C)

‘a’ = коэффициент сопротивления материала проводника

T = Температура проводника в ° C

T_ref = Эталонная температура, которая указана для проводника «a» материал.

===

Температурный коэффициент:

Температура проводника может сильно повлиять на его сопротивление. === приведены значения Ом-на-мил-фут (K) при 20 ° C для различных материалов. Сопротивление материала обычно дается при 20 ° C, потому что это стандарт, используемый в Справочник американских инженеров и считается стандартом во всем Соединенные Штаты. Температурный коэффициент можно использовать для определения стойкость материала при разных температурах.Большинство дирижеров будут повышают свою стойкость при повышении температуры. Полупроводник материалы, такие как кремний, германий и углерод, будут демонстрировать снижение сопротивления при повышении температуры. Эти материалы имеют отрицательный коэффициент температуры.

+++++ Однофазный проводник вызывает выделение тепла в канале. Conduit Жилет однофазный.

+++++ Магнитное поле расширяется и сжимается. Магнитный проводник поле.

+++++ Вихревые токи — это токи, индуцированные в металлах.

Параллельные проводники

При определенных условиях может потребоваться или выгодно подключить проводники параллельно. Одно из таких условий для параллельных проводников — это когда проводник очень большой, как в предыдущем примере, где он был рассчитан что проводники, питающие двигатель на расстоянии 2500 футов от источника, будут иметь быть 500 тыс. куб. Проводник на 500 тысяч кубометров очень большой и сложный в обращении.Поэтому для этой установки может быть предпочтительнее использовать параллельные проводники. NEC перечисляет пять условий, которые должны выполняться при подключении проводов. параллельно (310.10 (H)). Эти условия перечислены здесь:

1. Жилы должны быть одинаковой длины.

2. Жилы должны быть из одного материала. Итак, все параллельные проводники должен быть либо медным, либо алюминиевым. Запрещается использовать медь для один проводник, а другой — алюминий.

3. Проводники должны иметь одинаковую площадь в миллиметрах.

4. Проводники должны иметь одинаковую изоляцию.

5. Проводники должны быть заделаны или подключены таким же образом.

В этом примере фактический размер необходимого проводника был рассчитан как 440778,443. СМ. Эта круглая площадь в миле может быть получена путем соединения двух 250-тысячных милов. параллельные проводники для каждой фазы или три 000 (3/0) проводников, включенных параллельно для каждой фазы.[Примечание: каждый проводник 000 (3/0) имеет площадь 167 800 см. Это всего 503 400 см.] Еще один пример того, когда это может быть необходимо подключать провода параллельно — это когда необходимо проложить проводники большого сечения в канале. Запрещается вводить проводники одной фазы в металлические трубы [NEC 300.5 (I), NEC 300.20 (A) и NEC 300.20 (B)], потому что когда ток течет по проводнику, вокруг создается магнитное поле. проводник. В цепи переменного тока ток постоянно меняет направление и величина, которая заставляет магнитное поле прорезать стенку металлический трубопровод.Это режущее действие магнитного поля индуцирует ток, называемый вихревым током, в металл кабелепровода. Вихревые токи бывают токи, которые индуцируются в металлах. Они имеют тенденцию двигаться по кругу похожи на водовороты реки, отсюда и название вихревые течения (). вихревые токи может выделять достаточно тепла в сильноточных цепях, чтобы расплавить окружающую изоляцию. проводники. Все металлические трубопроводы могут иметь индукцию вихревых токов, но трубопроводы изготовленные из магнитных материалов, таких как сталь, имеют дополнительную проблему с гистерезисом потеря.Потеря гистерезиса вызвана трением молекул. Как направление магнитного поля меняет направление, молекулы металла намагничиваются с противоположной полярностью и качните, чтобы перестроиться. Этот непрерывный выравнивание и перестройка молекул производит тепло, вызванное трением. Потери на гистерезис увеличиваются с увеличением частоты.

+++++ Молекулы меняют направление при каждом изменении магнитного поля. направление.

+++++ Каждый кабелепровод содержит проводник от каждой фазы.Это позволяет магнитные поля нейтрализуют друг друга.

Чтобы устранить эту проблему, в каждом кабелепроводе должен быть проложен провод каждой фазы. Когда все три фазы содержатся в одном трубопроводе, магнитные поля отдельных проводников компенсируют друг друга, в результате чего ток не протекает. индуцируется в стенках водовода.

+++++ MEGGER на батарейках.

+++++ Тестирование шорт с помощью MEGGER.

+++++ Тестирование грунта с помощью MEGGER

Тестирование проводов

После установки проводников в кабелепроводы или кабельные каналы допускается потренируйтесь проверить установку на заземление и шорты.Этот тест требует омметр, который не только измеряет сопротивление в миллионах Ом, но и обеспечивает достаточно высокое напряжение, чтобы гарантировать, что изоляция не сломается вниз, когда к проводникам приложено номинальное сетевое напряжение. Большинство омметров работать с максимальным напряжением от 1,5 до 9 вольт в зависимости от типа омметра и настройки шкалы диапазонов. К Для проверки изоляции проводов используется специальный тип омметра, называемый MEGGER. MEGGER — это мегомметр, который может выдавать напряжения в диапазоне от примерно От 250 до 5000 вольт в зависимости от модели счетчика и настройки диапазона.

Показана одна модель MEGGER. Этот инструмент содержит рукоятку, которая подключен к ротору бесщеточного генератора переменного тока. Преимущество этого Особенность прибора заключается в том, что он не требует использования батареек. А Переключатель диапазонов позволяет использовать измеритель в качестве стандартного омметра. или как мегомметр. Когда он используется в качестве мегомметра, селекторный переключатель позволяет выбрать испытательное напряжение. Испытательные напряжения 100 вольт, 250 могут быть получены вольт, 500 вольт и 1000 вольт.

MEGGER также можно приобрести в моделях с батарейным питанием. Эти модели маленький, легкий и особенно полезный, когда возникает необходимость проверить диэлектрик конденсатора.

Провода обычно проверяются на наличие двух условий: короткое замыкание и заземление. Короткие замыкания — это пути тока, которые существуют между проводниками. Чтобы протестировать установку в случае коротких замыканий MEGGER подключается одновременно к двум проводам. В цепь испытывается при номинальном напряжении или немного выше.MEGGER указывает сопротивление между двумя проводниками.

Поскольку оба провода изолированы, сопротивление между ними должно быть чрезвычайно высоким. Каждый провод должен быть протестирован относительно любого другого проводника. в установке.

Для проверки установки на массу подключается один вывод MEGGER. к кабелепроводу или кабельному каналу. Другой провод измерителя подключается к одному из проводники. Провод следует испытывать при номинальном напряжении или немного выше.Каждый проводник следует проверить.

Теория цепей

— Что не так с параллельными проводниками?

Что касается схемы на втором рисунке, если каждый провод на схеме достаточно большой, чтобы безопасно выдерживать полный ток, требуемый нагрузкой, тогда схема не представляет опасности возгорания или перегрузки.

Рассмотрим схему (1):

Где провода 1 достаточно для зажигания лампы без перегрева.

Рассмотрим отдельно эту схему (2):

Где провода 2 также достаточно, чтобы без проблем зажечь лампу.

Теперь, начиная с контура 2, добавляем провод из контура 1:

Что происходит с током в проводе 2?

Ток в проводе 2 не может увеличиваться, потому что не добавляется дополнительный путь между ЛИНИЕЙ и A, а также между B и НАГРУЗКОЙ. Фактически, ток уменьшится, хотя маловероятно, что он уменьшится вдвое, если сопротивление двух проводов случайно не совпадет.

Теперь, начиная с контура 1, добавьте провод от контура 2.Что происходит с током в проводе 1? Он уменьшится, хотя — опять же — маловероятно, что он уменьшится вдвое.

Весь ток, требуемый нагрузкой, будет делиться между проводами 1 и 2, в зависимости от их сравнительного сопротивления, но ни один из проводов не будет пропускать больше всего тока. Поскольку любой из проводов может безопасно пропускать весь ток, опасность перегрузки отсутствует.

В качестве другого мысленного эксперимента начните с подсоединения обоих проводов и постепенно увеличивайте сопротивление провода 2.Что происходит с током в проводе 1? Он постепенно приближается к току полной нагрузки, но никогда не превышает его. Увеличьте сопротивление провода 2 до бесконечности, отрезав или удалив его, и ток в проводе 1 достигнет ровно тока полной нагрузки.

Пока выполняется условие, что провод 1 или провод 2 может безопасно питать нагрузку, не существует комбинации асимметричного сопротивления, которая приведет к перегрузке по току в любой части цепи. Вот почему схема на рисунке 2 не представляет опасности перегрева.

резисторов, подключенных параллельно — обмен электротехническими стеками

Несколько сопротивлений, включенных параллельно, для каждого потребляемого тока, а эффективное сопротивление — это сопротивление, которое потребляет такой же ток, как и комбинированные сопротивления.


Рассмотрим следующую версию закона Ома

Если у вас есть «черный ящик» с двумя подключенными проводами и вам сказали, что внутри есть резистор, вы можете измерить приложенное напряжение и потребляемый ток, чтобы определить внутреннее сопротивление.

Если вы подаете 10 В и на выходе 1 мА, вы заключаете, что \ $ R = \ dfrac {V} {I} = \ dfrac {10} {0.001} = 10k \ Omega \ $.

Теперь представьте, что внутри ДВА резистора и они включены параллельно. Снова подайте 10 В, и вы увидите, что потребляется 2 мА (а не 1 мА, как раньше). 1 мА проходит через 1 резистор, а 1 мА проходит через другой резистор.

Снаружи вы видите \ $ R = \ dfrac {V} {I} = \ dfrac {10} {0.002} = 5,000 \ Omega \ $.
Поскольку то, что находится внутри, потребляет тот же ток, что и \ $ 5000 \ Omega \ $, вы знаете, что внутри есть \ $ 5000 \ Omega \ $ или что-то с эквивалентным сопротивлением.Очевидно, что \ $ 2 \ times 10,000 \ Omega \ $ при параллельном подключении имеет сопротивление \ $ 5000 \ Omega \ $.

, т. Е. Несколько параллельных сопротивлений для каждого потребляемого тока, а эффективное сопротивление — это сопротивление, которое потребляет такой же ток, как и комбинированные сопротивления.

\ $ i_1 = \ dfrac {V} {R_1} \

$

\ $ i_2 = \ dfrac {V} {R_2} \

$

\ $ i_3 = \ dfrac {V} {R_3} \

$

\ $ i_ {total} = i_1 + i_2 + i_3 = V \ times \ Big (\ dfrac {1} {R_1} + \ dfrac {1} {R_2} + \ dfrac {1} {R_3} \ Big) \

долларов США

\ $ R_ {эффективный} = \ dfrac {V} {I_ {total}} \ $

Итак, \ $ I_ {total} = \ dfrac {V} {R_ {effective}} = V \ times (\ dfrac {1} {R_1} + \ dfrac {1} {R_2} + \ dfrac {1} {R_3}) \

$

, поэтому \ $ R_ {effective} = \ dfrac {1} {\ dfrac {1} {R_1} + \ dfrac {1} {R_2} + \ dfrac {1} {R_3}} \ $

Эффективное сопротивление = обратная сумма обратных сопротивлений.

например, 100 Ом + 200 Ом параллельно

\ $ R_ {эффективный} = \ dfrac {1} {\ dfrac {1} {100} + {1} {200}} = \ dfrac {1} {0,01 + 0,05} = \ dfrac {1} {0.015} = 66.666 \ Omega \ $

21.1 Последовательные и параллельные резисторы — BCIT Physics 0312 Учебник

На рисунке 3 показаны резисторы параллельно , подключенные к источнику напряжения. Резисторы включены параллельно, когда каждый резистор подключен непосредственно к источнику напряжения с помощью соединительных проводов с незначительным сопротивлением.Таким образом, к каждому резистору приложено полное напряжение источника.

Каждый резистор потребляет такой же ток, как если бы он один был подключен к источнику напряжения (при условии, что источник напряжения не перегружен). Например, автомобильные фары, радио и т. Д. Подключены параллельно, так что они используют полное напряжение источника и могут работать полностью независимо. То же самое и в вашем доме, или в любом другом здании. (См. Рис. 3 (b).)

Чтобы найти выражение для эквивалентного параллельного сопротивления [латекс] \ boldsymbol {R _ {\ textbf {p}}} [/ latex], давайте рассмотрим протекающие токи и то, как они связаны с сопротивлением.Поскольку каждый резистор в цепи имеет полное напряжение, токи, протекающие через отдельные резисторы, составляют [латекс] \ boldsymbol {I_1 = \ frac {V} {R_1}} [/ latex], [латекс] \ boldsymbol {I_2 = \ frac {V} {R_2}} [/ latex] и [latex] \ boldsymbol {I_3 = \ frac {V} {R_3}} [/ latex]. Сохранение заряда подразумевает, что общий ток [латекс] \ boldsymbol {I} [/ latex], производимый источником, является суммой этих токов:

[латекс] \ boldsymbol {I =} [/ латекс] [латекс] \ boldsymbol {\ frac {V} {R_1}} [/ latex] [латекс] \ boldsymbol {+} [/ latex] [латекс] \ boldsymbol {\ frac {V} {R_2}} [/ latex] [латекс] \ boldsymbol {+} [/ latex] [латекс] \ boldsymbol {\ frac {V} {R_3}} [/ latex] [латекс] \ boldsymbol {= V} [/ latex] [латекс] \ boldsymbol {(\ frac { 1} {R_1}} [/ latex] [латекс] \ boldsymbol {+} [/ latex] [латекс] \ boldsymbol {\ frac {1} {R_2}} [/ latex] [латекс] \ boldsymbol {+} [ / latex] [латекс] \ boldsymbol {\ frac {1} {R_3})}.[/ latex]

[латекс] \ boldsymbol {I =} [/ latex] [латекс] \ boldsymbol {\ frac {V} {R_p}} [/ latex] [латекс] \ boldsymbol {= V} [/ латекс ] [latex] \ boldsymbol {(\ frac {1} {R_p})}. [/ latex]

Члены в скобках в последних двух уравнениях должны быть равны. Обобщая для любого количества резисторов, общее сопротивление [латекс] \ boldsymbol {R_p} [/ latex] параллельного соединения связано с отдельными сопротивлениями соотношением

[латекс] \ boldsymbol {\ frac {1} {R_p}} [/ latex] [латекс] \ boldsymbol {=} [/ latex] [латекс] \ boldsymbol {\ frac {1} {R_1}} [/ latex] [латекс] \ boldsymbol {+} [/ latex] [латекс] \ boldsymbol {\ frac {1} {R_2}} [/ latex] [латекс] \ boldsymbol {+} [/ latex] [латекс] \ boldsymbol {\ frac {1} {R_3}} [/ latex] [латекс] \ boldsymbol {+ \ cdots} [/ latex]

Это соотношение приводит к общему сопротивлению [латекс] \ boldsymbol {R_p} [/ latex], которое меньше наименьшего из отдельных сопротивлений.(Это видно в следующем примере.) При параллельном подключении резисторов от источника течет больше тока, чем протекает по любому из них по отдельности, поэтому общее сопротивление ниже.

Пример 2: Расчет сопротивления, тока, рассеиваемой мощности и выходной мощности: анализ параллельной цепи

Пусть выходное напряжение батареи и сопротивления в параллельном соединении на Рисунке 3 будут такими же, как и в ранее рассмотренном последовательном соединении: [latex] \ boldsymbol {V = 12.0 \; \ textbf {V}} [/ latex], [latex] \ boldsymbol {R_1 = 1.00 \; \ Omega} [/ latex], [latex] \ boldsymbol {R_2 = 6.00 \; \ Omega} [/ латекс ] и [латекс] \ boldsymbol {R_3 = 13.0 \; \ Omega} [/ latex]. а) Каково полное сопротивление? (б) Найдите полный ток. (c) Рассчитайте токи в каждом резисторе и покажите, как они складываются, чтобы равняться общему выходному току источника. (d) Рассчитайте мощность, рассеиваемую каждым резистором. (e) Найдите выходную мощность источника и покажите, что она равна общей мощности, рассеиваемой резисторами.

Стратегия и решение для (а)

Общее сопротивление для параллельной комбинации резисторов находится с помощью следующего уравнения. Ввод известных значений дает

[латекс] \ boldsymbol {\ frac {1} {R_p}} [/ latex] [латекс] \ boldsymbol {=} [/ latex] [латекс] \ boldsymbol {\ frac {1} {R_1}} [/ латекс ] [латекс] \ boldsymbol {+} [/ латекс] [латекс] \ boldsymbol {\ frac {1} {R_2}} [/ latex] [латекс] \ boldsymbol {+} [/ латекс] [латекс] \ boldsymbol { \ frac {1} {R_3}} [/ latex] [latex] \ boldsymbol {=} [/ latex] [latex] \ boldsymbol {\ frac {1} {1.00 \; \ Omega}} [/ latex] [латекс] \ boldsymbol {+} [/ latex] [latex] \ boldsymbol {\ frac {1} {6.00 \; \ Omega}} [/ latex] [латекс] \ boldsymbol {+} [/ latex] [латекс] \ boldsymbol {\ frac {1} {13.0 \; \ Omega}}. [/ latex]

Таким образом,

[латекс] \ boldsymbol {\ frac {1} {R_p}} [/ latex] [латекс] \ boldsymbol {=} [/ latex] [латекс] \ boldsymbol {\ frac {1.00} {\ Omega}} [/ латекс] [латекс] \ boldsymbol {+} [/ латекс] [латекс] \ boldsymbol {\ frac {0.1667} {\ Omega}} [/ латекс] [латекс] \ boldsymbol {+} [/ латекс] [латекс] \ boldsymbol {\ frac {0.07692} {\ Omega}} [/ latex] [латекс] \ boldsymbol {=} [/ latex] [латекс] \ boldsymbol {\ frac {1.2436} {\ Omega}} [/ латекс]

(Обратите внимание, что в этих расчетах каждый промежуточный ответ отображается с дополнительной цифрой.)

Мы должны перевернуть это, чтобы найти полное сопротивление [латекс] \ boldsymbol {R_p} [/ latex]. Это дает

[латекс] \ boldsymbol {R_p =} [/ latex] [латекс] \ boldsymbol {\ frac {1} {1.2436}} [/ latex] [латекс] \ boldsymbol {\ Omega = 0.8041 \; \ Omega}. [ / латекс]

Общее сопротивление с правильным количеством значащих цифр составляет [латекс] \ boldsymbol {R_p = 0.804 \; \ Omega} [/ latex]

Обсуждение для (а)

[латекс] \ boldsymbol {R_p} [/ latex], как и предполагалось, меньше наименьшего индивидуального сопротивления.

Стратегия и решение для (b)

Полный ток можно найти из закона Ома, заменив полное сопротивление [латекс] \ boldsymbol {R_p} [/ latex]. Это дает

[латекс] \ boldsymbol {I =} [/ latex] [латекс] \ boldsymbol {\ frac {V} {R_p}} [/ latex] [латекс] \ boldsymbol {=} [/ latex] [латекс] \ boldsymbol {\ frac {12.0 \; \ textbf {V}} {0.8041 \; \ Omega}} [/ latex] [latex] \ boldsymbol {= 14.92 \; \ textbf {A}} [/ latex]

Обсуждение для (б)

Ток [latex] \ boldsymbol {I} [/ latex] для каждого устройства намного больше, чем для тех же устройств, подключенных последовательно (см. Предыдущий пример).Схема с параллельным соединением имеет меньшее общее сопротивление, чем резисторы, включенные последовательно.

Стратегия и решение для (c)

Отдельные токи легко вычислить по закону Ома, поскольку каждый резистор получает полное напряжение. Таким образом,

[латекс] \ boldsymbol {I_1 =} [/ latex] [латекс] \ boldsymbol {\ frac {V} {R_1}} [/ latex] [латекс] \ boldsymbol {=} [/ latex] [латекс] \ boldsymbol {\ frac {12.0 \; \ textbf {V}} {1.00 \; \ Omega}} [/ latex] [latex] \ boldsymbol {= 12.0 \; \ textbf {A}}. [/ Latex]

Аналогично

[латекс] \ boldsymbol {I_2 =} [/ latex] [латекс] \ boldsymbol {\ frac {V} {R_2}} [/ latex] [латекс] \ boldsymbol {=} [/ latex] [латекс] \ boldsymbol {\ frac {12.0 \; \ textbf {V}} {6.00 \; \ Omega}} [/ latex] [latex] \ boldsymbol {= 2.00 \; \ textbf {A}} [/ latex]

и

[латекс] \ boldsymbol {I_3 =} [/ latex] [латекс] \ boldsymbol {\ frac {V} {R_3}} [/ latex] [латекс] \ boldsymbol {=} [/ latex] [латекс] \ boldsymbol {\ frac {12.0 \; \ textbf {V}} {13.0 \; \ Omega}} [/ latex] [latex] \ boldsymbol {= 0.92 \; \ textbf {A}}. [/ Latex]

Обсуждение для (c)

Общий ток складывается из отдельных токов:

[латекс] \ boldsymbol {I_1 + I_2 + I_3 = 14.92 \; \ textbf {A}}. [/ Latex]

Это соответствует сохранению заряда.

Стратегия и решение для (d)

Мощность, рассеиваемую каждым резистором, можно найти с помощью любого из уравнений, связывающих мощность с током, напряжением и сопротивлением, поскольку все три известны.2} {13.0 \; \ Omega}} [/ latex] [latex] \ boldsymbol {= 11.1 \; \ textbf {W}}. [/ Latex]

Обсуждение для (д)

Мощность, рассеиваемая каждым резистором при параллельном подключении, значительно выше, чем при последовательном подключении к тому же источнику напряжения.

Стратегия и решение для (e)

Общую мощность также можно рассчитать несколькими способами. Выбирая [латекс] \ boldsymbol {P = IV} [/ latex] и вводя общий ток, получаем

[латекс] \ boldsymbol {P = IV = (14.92 \; \ textbf {A}) (12.0 \; \ textbf {V}) = 179 \; \ textbf {W}}. [/ Latex]

Обсуждение для (e)

Общая мощность, рассеиваемая резисторами, также составляет 179 Вт:

[латекс] \ boldsymbol {P_1 + P_2 + P_3 = 144 \; \ textbf {W} + 24.0 \; \ textbf {W} + 11.1 \; \ textbf {W} = 179 \; \ textbf {W}}. [/ латекс]

Это соответствует закону сохранения энергии.

Общее обсуждение

Обратите внимание, что как токи, так и мощность при параллельном подключении больше, чем для тех же устройств, подключенных последовательно.

2 параллельных провода

Относительная проницаемость \ ({\ mu _r} \) во всех области в окрестности двух параллельных проводников влияют на расчет индуктивности. Сюда входит стоимость \ ({\ mu _r} \) из сами проводники, \ ({\ mu _r} \) вне проводников, а если они полые, значение из \ ({\ mu _r} \) в интерьер проводников.

Многие электрические изоляторы имеют \ ({\ mu _r} \) очень близко к одному. Это включает в себя вакуум, почти все газы, стекло и многое другое. керамические материалы и многие электрические изоляторы из продуктов нефтехимии.Если проводники полые, то в расчетах на этой странице предполагается, что внутри проводников был изолятор с \ ({\ mu _r} \) одного. Обратите внимание, что в некоторых проводниках используется стальной сердечник с символом \ ({\ mu _r} \) значительно больше единицы. Расчеты на этой странице не решают эту проблему. Добрый дирижера.

Число электрических проводников переменного тока может значительно увеличиться в их стойкость за счет скин-эффекта. Это можно свести к минимуму, выбрав проводники. который имеет а \ ({\ mu _r} \) одного. Многие часто используемые проводники, в том числе алюминий и медь имеют \ ({\ mu _r} \) одного, как и такие материалы, как золото и вода.Сплавы на основе железа, такие как сталь, являются заметным исключение, с \ ({\ mu _r} \), которое может проникнуть в тысячи и выше. Расчеты на этой странице предполагают, что проводник имеет \ ({\ mu _r} \) одного.

\ ({\ mu _r} \) во всех областях вокруг проводников влияют на их индуктивность.В приведенных выше расчетах предполагается, что \ ({\ mu _r} \) была единообразной во всех регионах возле проводников.

Многие инструменты, машины, офисная мебель, корпуса и здания изготавливаются с использованием сплавы на основе железа, такие как сталь.Обратите внимание, что индуктивность параллельного набора проводников будет сильно изменена. по проезжая мимо этих объектов.

Резисторы

в последовательном и параллельном выводе формулы »Электроника

Производная от формул для расчета полного сопротивления последовательно включенных резисторов и параллелей.


Resistance Tutorial:
Что такое сопротивление Закон Ома Омические и неомические проводники Сопротивление лампы накаливания Удельное сопротивление Таблица удельного сопротивления для распространенных материалов Температурный коэффициент сопротивления Электрическая проводимость Последовательные и параллельные резисторы Таблица параллельных резисторов


Формулы для расчета полного сопротивления для ряда резисторов, включенных последовательно, а также для резисторов, включенных параллельно, хорошо известны.

Менее известны обоснование и вывод формул.

Понимание того, как вывести формулы для набора резисторов, включенных последовательно или параллельно, может потребоваться в некоторых случаях, а также помогает в понимании общей теории схем.

В основе вывода уравнений для последовательного и параллельного резисторов лежит использование законов Кирхгофа. Используя их, вывод уравнений относительно прост.

Определение общего сопротивления резисторов в серии

Уравнение для полного сопротивления серии резисторов, включенных параллельно, представляет собой сумму всех резисторов, приведенных ниже.

Резисторы последовательно

Первый этап доказательства формулы — рассмотреть случай с двумя последовательно включенными резисторами, чтобы увидеть, как ведет себя схема.

Последовательные резисторы — отдельные резисторы, токи и напряжения
Здесь показаны два резистора, но одно и то же соединение может быть легко расширено до любого количества резисторов.

При выводе уравнения для полного сопротивления набора последовательно включенных резисторов необходимо учитывать два факта. Во-первых, по цепи течет один и тот же ток. Одинаковый ток протекает через источник напряжения и через резисторы.

Во-вторых, законы Кирхгофа гласят, что сумма напряжений в цепи равна нулю. Таким образом, сумма падений напряжения на резисторах равна напряжению, подаваемому источником в показанной цепи.

Из закона Ома:

V1 = IR1 & V2 = IR2

Тогда из закона Кирхгофа:

V-V1-V2 = 0 или V = V1 + V2

Затем замена на V 1 и V 2

V = IR1 + IR2 = I (R1 + R2)

Это упрощается до:

VI = R1 + R2

Но V / I = R всего , следовательно,

Rtotal = R1 + R2

Используя ту же логику, можно расширить это до общего случая нескольких резисторов:

Rtotal = R1 + R2 + R3 + ….

Определение общего сопротивления резисторов, включенных параллельно

Часто бывает так, что несколько резисторов устанавливаются параллельно.Это происходит во многих случаях при проектировании электронных схем и т. Д.

Стандартная формула для расчета общего сопротивления для ряда резисторов или опорных элементов, включенных параллельно, приведена ниже.

1Rtotal = 1R1 + 1R2 + 1R3 + ……

Вывод общего уравнения для набора нескольких параллельно включенных резисторов довольно легко выполнить. Взяв основные аспекты схемы, можно легко вывести общее уравнение для набора резисторов, включенных параллельно.

Параллельные резисторы — отдельные резисторы, токи и напряжения

При выводе формулы для общего сопротивления набора резисторов, включенных параллельно, необходимо учитывать ток, протекающий через каждый резистор по очереди, и понимать, что каждый резистор имеет такая же разность потенциалов или напряжение на нем.

Первое, что нужно понять, это то, что сумма токов, протекающих через отдельные резисторы, равна общему току, обеспечиваемому источником напряжения, как показано на диаграмме: Я = I1 + I2 + I3 +…In

Зная, что I = V / R из закона Ома, можно связать текущие уровни тока с точки зрения напряжения (которое одинаково для всех, поскольку они параллельны), и сопротивления.

I = VR1 + VR2 + VR3 + … VRn

Тогда, разделив обе стороны на V, мы можем увидеть:

IV = 1R1 + 1R2 + 1R3 + … 1Rn

Но поскольку I / V составляет всего 1 / R , это можно заменить в уравнении, чтобы получить:

1Rtotal = 1R1 + 1R2 + 1R3 + … 1Rn

Видно, что вывод для полного сопротивления серии резисторов, включенных параллельно, очень легко получить.

Вывод формулы для двух резисторов, включенных параллельно

Часто бывает, что в различных электрических и электронных схемах или установках необходимо вычислить общее сопротивление для двух параллельно подключенных резисторов.

В этом случае уравнение можно значительно упростить, что значительно упростит расчет полного сопротивления.

Вывести это уравнение относительно просто, требуя некоторых простых манипуляций с общим уравнением для параллельных резисторов, но упрощая его до включения только двух электронных компонентов.

1Rtotal = 1R1 + 1R2

Умножение на Ral дает:

1 = RtotalR1 + RtR2

Затем умножить на 1 и 2

R1R2 = RtotalR2 + RtotalR1

Изолировать R всего R1R2 = Rtotal (R2 + R1)

Затем разделите на ( 1 + R 2 )

Rtotal = R1R2R2 + R1

Используя эту формулу, очень легко рассчитать общее сопротивление двух резисторов, включенных параллельно

Уравнения для определения общего сопротивления для наборов резисторов, включенных последовательно и параллельно, широко используются во многих областях, от электромонтажных работ до проектирования электронных схем и во многих других областях.Хотя нет необходимости постоянно выводить уравнения из первых принципов, полезно понять, как это можно сделать, поскольку это дает гораздо лучшее понимание того, что происходит.

Дополнительные концепции и учебные пособия по основам электроники:
Voltage Текущий Мощность Сопротивление Емкость Индуктивность Трансформеры Децибел, дБ Законы Кирхгофа Q, добротность РЧ шум
Вернуться в меню «Основные понятия электроники».. .

ПАРАЛЛЕЛЬНЫЕ ЦЕПИ

ПАРАЛЛЕЛЬНЫЕ ЦЕПИ ПАРАЛЛЕЛЬНЫЕ ЦЕПИ ПОСТОЯННОГО ТОКА

Цепь, в которой два или более электрических сопротивления или нагрузки подключен к одному источнику напряжения и представляет собой параллельную цепь. Параллель Схема отличается от последовательной схемы тем, что предусмотрено более одного пути для текущего потока — чем больше путей добавлено параллельно, тем меньше противодействия к потоку электронов от источника.В последовательной схеме сложение сопротивления увеличивает сопротивление току. Минимальные требования для параллельной цепи следующие:

(1) источник питания.
(2) проводника.
(3) сопротивление или нагрузка для каждого пути тока.
(4) два или более путей для прохождения тока.

На рисунке 8-62 показана параллельная цепь с тремя путями прохождения тока. Точки A, B, C и D подключены к одному и тому же проводнику и находятся на такой же электрический потенциал.Аналогичным образом точки E, F, G и H с таким же потенциалом. Поскольку приложенное напряжение появляется между точками A и E, одинаковое напряжение приложено между точками B и F, точками C и G, а между точками D и H. Таким образом, при параллельном включении резисторов на источнике напряжения каждый резистор имеет одинаковое приложенное напряжение, хотя токи через резисторы могут отличаться в зависимости от значений сопротивление. Напряжение в параллельной цепи можно выразить следующим образом:

Где ЕТ — приложенное напряжение, Е1 — напряжение на R1, Е2 — напряжение на R2, а E3 — это напряжение на R3 (рисунок 8-62).

Ток в параллельной цепи делится между различными ветвями. в зависимости от сопротивления каждой ветви (см. рисунок 8-63). Ветвь с малым сопротивлением будет иметь больший ток. поток, чем ветвь, содержащая высокое сопротивление. Действующий закон Кирхгофа утверждает, что ток, текущий к точке, равен току утекает из этой точки.Таким образом, ток в цепи может быть математически выражается следующим образом:
где IT — общая ток, а I1, I2 и I3 — токи через R1, R2 и R3 соответственно. Закон Кирхгофа и Ома можно применить, чтобы найти полный ток в схеме, показанной на рисунке 8-63.

Ток через ветвь, содержащую сопротивление R1, составляет

Ток через R2 равен

Ток через R3 равен

Суммарный ток IT равен

В параллельной цепи IT = I1 + I2 + I3.По закону Ома следующее отношения можно получить:

Подставляя эти значения в уравнение для полного тока,

В параллельной цепи ET = E1 = E2 = E3. Следовательно,

Деление на E дает,

Это уравнение является обратной формулой для нахождения общей суммы или ее эквивалента. сопротивление параллельной цепи. Другая форма уравнения может быть получена решая для RT.

Анализ уравнения для полного сопротивления в параллельной цепи показывает, что RT всегда меньше наименьшего сопротивления в параллельном схема. Таким образом, резисторы на 10 Ом, 20 Ом и 40 Ом подключены параллельно. иметь общее сопротивление менее 10 Ом.

Если в параллельной цепи всего два резистора, обратная формула

В упрощенном виде это становится:

Эта упрощенная, более короткая формула может использоваться, когда два сопротивления в параллели.Другой метод можно использовать для любого количества резисторов в параллельны, если они имеют одинаковое сопротивление. Величина сопротивления одного резистора делится на количество параллельно включенных резисторов, чтобы определить общую сопротивление.

Провод

Добавить комментарий

Ваш адрес email не будет опубликован. Обязательные поля помечены *